www.vorkurse.de
Ein Projekt von vorhilfe.de
Die Online-Kurse der Vorhilfe

E-Learning leicht gemacht.
Hallo Gast!einloggen | registrieren ]
Startseite · Mitglieder · Teams · Forum · Wissen · Kurse · Impressum
Forenbaum
^ Forenbaum
Status Mathe-Vorkurse
  Status Organisatorisches
  Status Schule
    Status Wiederholung Algebra
    Status Einführung Analysis
    Status Einführung Analytisc
    Status VK 21: Mathematik 6.
    Status VK 37: Kurvendiskussionen
    Status VK Abivorbereitungen
  Status Universität
    Status Lerngruppe LinAlg
    Status VK 13 Analysis I FH
    Status Algebra 2006
    Status VK 22: Algebra 2007
    Status GruMiHH 06
    Status VK 58: Algebra 1
    Status VK 59: Lineare Algebra
    Status VK 60: Analysis
    Status Wahrscheinlichkeitst

Gezeigt werden alle Foren bis zur Tiefe 2

Navigation
 Startseite...
 Neuerdings beta neu
 Forum...
 vorwissen...
 vorkurse...
 Werkzeuge...
 Nachhilfevermittlung beta...
 Online-Spiele beta
 Suchen
 Verein...
 Impressum
Das Projekt
Server und Internetanbindung werden durch Spenden finanziert.
Organisiert wird das Projekt von unserem Koordinatorenteam.
Hunderte Mitglieder helfen ehrenamtlich in unseren moderierten Foren.
Anbieter der Seite ist der gemeinnützige Verein "Vorhilfe.de e.V.".
Partnerseiten
Weitere Fächer:

Open Source FunktionenplotterFunkyPlot: Kostenloser und quelloffener Funktionenplotter für Linux und andere Betriebssysteme
Forum "Integrationstheorie" - Differenzierbar?!
Differenzierbar?! < Integrationstheorie < Maß/Integrat-Theorie < Analysis < Hochschule < Mathe < Vorhilfe
Ansicht: [ geschachtelt ] | ^ Forum "Integrationstheorie"  | ^^ Alle Foren  | ^ Forenbaum  | Materialien

Differenzierbar?!: Frage (beantwortet)
Status: (Frage) beantwortet Status 
Datum: 15:43 Fr 25.05.2007
Autor: cutter

Aufgabe
Sei f: [0,1]-> [mm] \IR [/mm] eine Lebesgue integr. Fkt. Muss die Fkt
[mm] F(x)=\integral_{0}^{x}{f(t) d\lambda(t)} [/mm] differenzierbar sein?

Ich denke nicht,dass das stimmt, jedoch faellt mir spontan keine Fkt ein. Hat einer eine Idee ?
Liebe Grüße :)

        
Bezug
Differenzierbar?!: Idee
Status: (Antwort) fertig Status 
Datum: 15:53 Fr 25.05.2007
Autor: generation...x

Unabhängig davon, was du denkst: Hast du mal versucht, den Differentialquotienten von F zu bilden? Versuch das mal...

Bezug
                
Bezug
Differenzierbar?!: Frage (beantwortet)
Status: (Frage) beantwortet Status 
Datum: 16:05 Fr 25.05.2007
Autor: cutter

Hi

[mm] \lim_{h\rightarrow 0} \frac{\integral_{}^{x+h}{f(t) d\lambda t}-\integral_{}^{x}{f(t) d\lambda t}}{h} [/mm]


das sollte ja der diffquotient sein ...aber viel mehr sieht man hier ja auch nciht ?:)
LG

Bezug
                        
Bezug
Differenzierbar?!: Antwort
Status: (Antwort) fertig Status 
Datum: 17:26 Fr 25.05.2007
Autor: generation...x

Doch, doch. Du musst nur etwas weiterrechnen und dann den Mittelwertsatz der Integration zur Abschätzung des Integrals verwenden (die zweite Funktion ist natürlich 1).
Ach, und h geht gegen 0 ...

Bezug
                                
Bezug
Differenzierbar?!: Frage (beantwortet)
Status: (Frage) beantwortet Status 
Datum: 14:18 Sa 26.05.2007
Autor: cutter

Hi ;)
deine idee finde ich ja ganz interessant ...aber mir fehlt der entscheidene Schritt die Integrale zusammen zufassen oder ueberhaupt den Quotienten zu vereinfachen....muss ich das erste integral irgendwie aufteilen ?

LG

Bezug
                                        
Bezug
Differenzierbar?!: Antwort
Status: (Antwort) fertig Status 
Datum: 01:12 So 27.05.2007
Autor: generation...x

Na gut, dann gaaanz langsam:

[mm] \lim_{h\rightarrow 0} \frac{\integral_{0}^{x+h}{f(t) d\lambda t}-\integral_{0}^{x}{f(t) d\lambda t}}{h} = \lim_{h\rightarrow 0} \frac{\integral_{x}^{x+h}{f(t) d\lambda t}}{h}[/mm]

Außerdem existiert nach Mittelwertsatz ein [mm] \xi \in [x, x+h] [/mm], so dass

[mm] \integral_{x}^{x+h}{f(t) d\lambda t} = f(\xi)\integral_{x}^{x+h}{ d\lambda t}[/mm]

Wenn man das einsetzt, das Integral ausrechnet (nicht so schwer...) und den Grenzwert bildet, ist man schon fertig.

Bezug
                                                
Bezug
Differenzierbar?!: Mitteilung
Status: (Mitteilung) Reaktion unnötig Status 
Datum: 12:09 So 27.05.2007
Autor: kornfeld


> [mm]\lim_{h\rightarrow 0} \frac{\integral_{0}^{x+h}{f(t) d\lambda t}-\integral_{0}^{x}{f(t) d\lambda t}}{h} = \lim_{h\rightarrow 0} \frac{\integral_{x}^{x+h}{f(t) d\lambda t}}{h}[/mm]
>  
> Außerdem existiert nach Mittelwertsatz ein [mm]\xi \in [x, x+h] [/mm],
> so dass
>
> [mm]\integral_{x}^{x+h}{f(t) d\lambda t} = f(\xi)\integral_{x}^{x+h}{ d\lambda t}[/mm]

Das ist nicht korrekt. Der Mittelwertsatz ist nicht auf Lebesgue-integrierbare Funktionen anwendbar! Zwar gibt es ein schwaches Konzept der Stetigkeit fuer solche Funktionen (die sogenannte approximative Stetigkeit von [mm] $L^1$-Funktionen [/mm] -> F.Morgan:Geometric measure theory for beginners, 1te Kapitel), Jedoch kann man nicht alle Resultate stetiger Funktionen auch auf solche "schwach" stetigen Funktionen uebertragen. Siehe mein Gegenbeispiel.

LG Kornfeld

Bezug
                                                        
Bezug
Differenzierbar?!: Mitteilung
Status: (Mitteilung) Reaktion unnötig Status 
Datum: 14:40 Mo 28.05.2007
Autor: generation...x

OK, stimmt auf einer Nullmenge - und [mm] \{ x \} [/mm] ist eine Nullmenge - kann f beliebig unschön sein. Die Frage ist, ob nicht trotzdem eine "schöne" Funktion [mm] \dot{F} [/mm] existiert, so dass

[mm] F(x) = \integral_{0}^{x}{\dot{F}(t) d\lambda t} [/mm]

[mm] \lim_{h\rightarrow 0} \integral_{x}^{x+h}{\dot{F}(t) d\lambda t} = \lim_{h\rightarrow 0} \dot{F}(x) h[/mm]

und [mm] \dot{F}(x) = f(x)[/mm] fast überall. Dieses [mm] \dot{F} [/mm] würde es ja tun.


Bezug
        
Bezug
Differenzierbar?!: Antwort
Status: (Antwort) fertig Status 
Datum: 16:04 Fr 25.05.2007
Autor: kornfeld

Ich habe mir das so gedacht. Nach dem Hauptsatz der Integral und Differentialrechnung ist fuer jedes stetige [mm] $f\in [/mm] C([0,1])$ $F$ eine Stammfunktion und diese differenzierbar. Das bedeutet folgendes: Wenn es ein $f$ gibt fuer welches $F$ nicht diffbar ist, dann muss dieses $f$ notwendig irgendwo unstetig sein. Ich habe dann ein bisschen mit [mm] $f(t)=\frac{1}{\sqrt{t}}$ [/mm] herumgespielt...

Lg Kornfeld

Bezug
                
Bezug
Differenzierbar?!: Frage (beantwortet)
Status: (Frage) beantwortet Status 
Datum: 16:21 Fr 25.05.2007
Autor: cutter

hi kornfeld:)

ich musste auch meinen artikel nochmal aendern...es war [mm] F(x)=\integral_{0}^{x}{f(t) d\lambda(t)} [/mm] wobei [mm] \lambda [/mm] das Lebesgue Maß ist.

mit [mm] f(t)=\frac{1}{\sqrt{t}} [/mm] komm ich bei ganz "normalem" bilden des DiffQuotienten auf..


.... [mm] -\frac{1}{\sqrt{x}} [/mm] also existiert der Grenzwert fuer x=0 nicht


Bezug
                        
Bezug
Differenzierbar?!: Antwort
Status: (Antwort) fertig Status 
Datum: 16:33 Fr 25.05.2007
Autor: kornfeld


> hi kornfeld:)
>  
> ich musste auch meinen artikel nochmal aendern...es war
> [mm]F(x)=\integral_{0}^{x}{f(t) d\lambda(t)}[/mm] wobei [mm]\lambda[/mm] das
> Lebesgue Maß ist.
>  
> mit [mm]f(t)=\frac{1}{\sqrt{t}}[/mm] komm ich bei ganz "normalem"
> bilden des DiffQuotienten auf..
>  
>
> .... [mm]-\frac{1}{\sqrt{x}}[/mm] also existiert der Grenzwert fuer
> x=0 nicht

Piano! Hier kannst du nicht einfach "hart" ableiten. Du solltest meine vorgeschlagene Funktion auch eventuell an das Intervall anpassen, z.b. [mm] $f(t)=\frac{1}{\sqrt{\vert t-1/2\vert}}$ [/mm] oder so aehnlich, damit nachher unmissverstaendlich ein $F$ herauskommt, das ueber dem offenen Intervall $(0,1)$ nicht diffbar ist. Um $F$ zu berechnen, akannst du partiell argumentieren (es kann ja keine diffbare Stammfunktion von $f$ geben)

LG Kornfeld

Bezug
                                
Bezug
Differenzierbar?!: Frage (beantwortet)
Status: (Frage) beantwortet Status 
Datum: 14:53 Mo 28.05.2007
Autor: cutter

Ok ...dann werde ich doch deinen Weg weiterverfolgen:)

Also muss ich eben eine Funktion konstruieren,die im intervall (0,1) unstetig ist.
Du hast ja
[mm] f(t)=\frac{1}{\sqrt{|t-\frac{1}{2}|}} [/mm] vorgeschlagen, die auch bei t=1/2 eine unsstetigkeitsstelle hat.

Wenn ich nun nicht integrieren soll, was meinst du denn dann mit partiell argumentieren ?

LG



Bezug
                                        
Bezug
Differenzierbar?!: Antwort
Status: (Antwort) fertig Status 
Datum: 15:59 Mo 28.05.2007
Autor: kornfeld


> Ok ...dann werde ich doch deinen Weg weiterverfolgen:)
>  
> Also muss ich eben eine Funktion konstruieren,die im
> intervall (0,1) unstetig ist.
>  Du hast ja
> [mm]f(t)=\frac{1}{\sqrt{|t-\frac{1}{2}|}}[/mm] vorgeschlagen, die
> auch bei t=1/2 eine unsstetigkeitsstelle hat.
>
> Wenn ich nun nicht integrieren soll, was meinst du denn
> dann mit partiell argumentieren ?

Nun ja, streng genommen, kann ja $f$ auf dem ganzen Intervall keine Stammfunktion besitzen, zumindest nicht in den Begrifflichkeiten der "klassischen" Differentialrechnung. Jedoch kann man dem Integral [mm] $\int_0^x \frac{dt}{\sqrt{\vert t-1/2\vert}}$ [/mm] durchaus einen Wert zuordnen. Naemlich so:
1) fuer $t$ zwischen $0$ und $1/2$ kennst du die Stammfunktion der Funktion [mm] $\frac{1}{\sqrt{\vert t-1/2\vert}}$ [/mm] und damit den Wert des obigen Integrals fuer $x$ zwischen $0$ und $1/2$.
2) Falls $x>1/2$ zerlegst du das Integral in zwei Integrale: eines in den Grenzen $0$ und $1/2$ und das andere in den Grenzen von $1/2$ und $x$. Das erste Integral kennst du bereits. Das zweite ist im Grunde wie jenes unter 1)...
Ueberzeuge dich, dass die Funktion $F(x)$ wirklich nicht differenzierbar in $1/2$ ist, indem du zeigst, dass der rechtsseitige Differenzenquotient keinen Limes in [mm] $\IR$ [/mm] hat.


LG Kornfeld

Bezug
                                                
Bezug
Differenzierbar?!: Frage (überfällig)
Status: (Frage) überfällig Status 
Datum: 16:51 Do 31.05.2007
Autor: cutter

Hi
So ich habe nun es so gemacht wie du gesagt hast:

1.) Fuer t zwischen 0, 1/2 folgt also
[mm] \int_0^{\frac{1}{2}} \frac{dt}{\sqrt{\vert t-1/2\vert}}=\sqrt{2} [/mm]

2.)
Fuer x> [mm] \frac{1}{2} [/mm] folgt
[mm] \int_0^x \frac{dt}{\sqrt{\vert t-1/2\vert}}=\sqrt{2}+\int_{\frac{1}{2}}^x \frac{dt}{\sqrt{\vert t-1/2\vert}} [/mm]

nun folgt aus dem Diffquotieten an der Stelle 1/2 ...ich lass mal die Wurzel 2 direkt Weg (kürzt sich ja weg)

[mm] F'(1/2)=\lim_{h->\infty}\frac{\int_{\frac{1}{2}}^{\frac{1}{2}+h} \frac{dt}{\sqrt{\vert t-1/2\vert}}}{h} [/mm]

Das ist dann ausgerechnet [mm] 2*\frac{(\vert h \vert)^{\frac{3}{2}}}{h^2} [/mm] und damit existiert der Limes nicht fuer h gegen 0.

Hab ichs nu richtig? LG  


Bezug
                                                        
Bezug
Differenzierbar?!: Fälligkeit abgelaufen
Status: (Mitteilung) Reaktion unnötig Status 
Datum: 17:20 Sa 02.06.2007
Autor: matux

$MATUXTEXT(ueberfaellige_frage)
Bezug
Ansicht: [ geschachtelt ] | ^ Forum "Integrationstheorie"  | ^^ Alle Foren  | ^ Forenbaum  | Materialien


^ Seitenanfang ^
www.vorkurse.de
[ Startseite | Mitglieder | Teams | Forum | Wissen | Kurse | Impressum ]